You are on page 1of 24

Question

If both 112 and 33 are factors of the number a * 43 * 62 * 1311, then what is the smallest possible
value of a?

1. 121
2. 3267
3. 363
4. 33

Correct choice (3). Correct Answer - (363)


Explanatory Answers

112 is a factor of the given number. The number does not have a power or multiple of 11 as its
factor. Hence, "a" should include 11 2

33 is a factor of the given number. 62 is a part of the number. 62 has 32 in it. Therefore, if 33 has to be
a factor of the given number a * 43 * 62 * 1311, then we will need at least another 3.

Therefore, if "a" should be at least 112 * 3 = 363 if the given number has to have 112 and 33 as its
factors.

Question
Find the greatest number of five digits, which is exactly divisible by 7, 10, 15, 21 and 28.

(1) 99840
(2) 99900
(3) 99960
(4) 99990

Correct Choice is (3) and the correct answer is 99960


Explanatory Answers

The number should be exactly divisible by 15 (3, 5), 21 (3, 7), 28 (4, 7).
Hence, it is enough to check the divisibility for 3, 4, 5 and 7.

99960 is the only number which satisfies the given condition.


Question 4 the day : August 11, 2003

The question for the day is from the topic Number Systems and involves operations on numbers.

Question
Anita had to do a multiplication. Instead of taking 35 as one of the multipliers, she took 53. As a
result, the product went up by 540. What is the new product?

(1) 1050
(2) 540
(3) 1440
(4) 1590

Correct choice is (4) and Correct Answer is 1590


Explanatory Answers

Let the number that Anita wanted to multiply be 'X'.


She was expected to find the value of 35X.
Instead, she found the value of 53X.

The difference between the value that she got (53X) and what she was expected to get (35X),
according to the question, is 540.

i.e., 53X - 35X = 540


or (53 - 35) * X = 540
X = 30
Therefore, new product = 53 * 30 = 1590
Question
Let x, y and z be distinct integers. x and y are odd and positive, and z is even and positive. Which
one of the following statements cannot be true?

(1) (x-z)2 y is even


(2) (x-z)y2 is odd
(3) (x-z)y is odd
(4) (x-y)2z is even

Correct Choice is (1) and Correct Answer is (x-z)2 y is even


Explanatory Answer

x and y are odd and positive and z is even and positive

(x - z)2 y is even cannot true

x - z is odd and y is odd


Therefore, (x - z)2 will be odd and (x - z)2 y will be odd
Question
When a number is divided by 36, it leaves a remainder of 19. What will be the remainder when the
number is divided by 12?

(1) 10
(2) 7
(3) 192
(4) None of these

Correct Choice is (2) and Correct Answer is 7


Explanatory Answer

Let the number be 'a'.

When 'a' is divided by 36, let the quotient be 'q' and we know the remainder is 19

i.e., and remainder is 19

or a = 36q + 19

when a is divided by 12, we get

or

36q is perfectly divided by 12

Therefore, remainder = 7
Question
The sum of the first 100 numbers, 1 to 100 is divisible by

(1) 2, 4 and 8
(2) 2 and 4
(3) 2 only
(4) None of these

Correct Choice is (3) and Correct Answer is 2 only


Explanatory Answer

The sum of the first 100 natural numbers is given by (n(n + 1))/2 = (100(101))/2 = 50(101).
101 is an odd number and 50 is divisible by 2. Hence, 50*101 will be divisible by 2.
Question
How many different factors are there for the number 48, excluding 1 and 48?

(1) 12
(2) 4
(3) 8
(4) None of these

Correct Choice is (3) and the correct answer is 8


Explanatory Answers

To find the number of factors of a given number, express the number as a product of powers of
prime numbers.

In this case, 48 can be written as 16 * 3 = (2 4 * 3)

Now, increment the power of each of the prime numbers by 1 and multiply the result.

In this case it will be (4 + 1)*(1 + 1) = 5 * 2 = 10 (the power of 2 is 4 and the power of 3 is 1)

Therefore, there will 10 factors including 1 and 48. Excluding, these two numbers, you will have 10 -
2 = 8 factors.
Question:
1025 - 7 is divisible by

1. 2
2. 3
3. 9
4. Both (2) and (3)

Correct Answer - 3. Correct choice is (2)


Explanatory Answers

1025 - 7 = (1025 - 1) - 6

The number 1025 - 1 = 99.....9 (25 digits) is divisible by 3 and 4.

Therefore, (1025 - 1) - 6 = (24 nines and unit digit is 3) 99.......93.

This number is only divisible by 3 (from the given choices).


Question
Find the G.C.D of 12x2y3z2, 18x3y2z4, and 24xy4z3

(1) 6xy2z2
(2) 6x3y4z3
(3) 24xy2z2
(4) 18x2y2z3

Correct Choice is (1) and Correct Answer is 6xy2z2


Explanatory Answer

G.C.D of 12, 18 and 24 is 6.

The common factors are x, y, z and their highest powers common to all are 1, 2 and 2 respectively.

Therefore, G.C.D = 6xy2z2

Question
What is the value of M and N respectively? If M39048458N is divisible by 8 and 11; Where M and N
are single digit integers?

(1) 7, 8
(2) 8, 6
(3) 6, 4
(4) 5, 4

Correct Choice is (3) and correct answer is 6, 4


Explanatory Answers

If the last three digits of a number is divisible by 8, then the number is divisible by 8 (test of
divisibility by 8).

Here, last three digits 58N is divisible by 8 if N = 4. (Since 584 is divisible by 8.)

For divisibility by 11. If the digits at odd and even places of a given number are equal or differ by a
number divisible by 11, then the given number is divisible by 11.

Therefore, (M+9+4+4+8)-(3+0+8+5+N)=(M+5) should be divisible by 11 => when M = 6.


Pe
Question:
A college has 10 basketball players. A 5-member team and a captain will be selected out of these 10
players. How many different selections can be made?

1. 1260
2. 210
3. 10C6 * 6!
4. 10C5 * 6

Correct Answer - 1260. Choice (1)


Explanatory Answer

A team of 6 members has to be selected from the 10 players. This can be done in 10C6 or 210 ways.
Now, the captain can be selected from these 6 players in 6 ways.
Therefore, total ways the selection can be made is 210*6 = 1260.

Alternatively, we can select the 5 member team out of the 10 in 10C5 ways = 252 ways.
The captain can be selected from amongst the remaining 5 players in 5 ways.
Therefore, total ways the selection of 5 players and a captain can be made = 252*5 = 1260.

Question:
Badri has 9 pairs of dark Blue socks and 9 pairs of Black socks. He keeps them all in a same bag.
If he picks out three socks at random what is the probability he will get a matching pair?

(1) (2*9C2 * 9
C1 ) / 18
C3
(2) (9C2 * 9
C1 ) / 18
C3
(3) 1
(4) None of these
Correct Answer - (3)
Solution:

If he picks any of the three socks invariably any two of them should match. Hence the
probability is 1.
Question:
How many words of 4 consonants and 3 vowels can be made from 12 consonants and 4 vowels,
if all the letters are different?
16
(1) C7 * 7!
12 4
(2) C4 * C3 * 7!
12 4
(3) C3 * C4
(4) 12C4 * 4C3
Correct Answer - (2)
Solution:
12
4 consonants out of 12 can be selected in C4 ways.
4
3 vowels can be selected in C3 ways.
12
Therefore, total number of groups each containing 4 consonants and 3 vowels = C4 * 4 C3

Each group contains 7 letters, which can be arranging in 7! ways.

Therefore required number of words = 124 * 4C3 * 7!

Question:
If the letters of the word CHASM are rearranged to form 5 letter words such that none of the
word repeat and the results arranged in ascending order as in a dictionary what is the rank of
the word CHASM?

(1) 24
(2) 31
(3) 32
(4) 30
Correct Answer - (3)
Solution:

The 5 letter word can be rearranged in 5! Ways = 120 without any of the letters repeating.

The first 24 of these words will start with A.

Then the 25th word will start will CA _ _ _. The remaining 3 letters can be rearranged in 3!
Ways = 6. i.e. 6 words exist that start with CA.

The next word starts with CH and then A, i.e., CHA _ _. The first of the words will be CHAMS.
The next word will be CHASM.

Therefore, the rank of CHASM will be 24 + 6 + 2 = 32.

Question:
How many four letter distinct initials can be formed using the alphabets of English language
such that the last of the four words is always a consonant?

(1) (26^3)*(21)
(2) 26*25*24*21
(3) 25*24*23*21
(4) None of these.
Correct Answer - (1)
Solution:

The last of the four letter words should be a consonant. Therefore, there are 21 options.

The first three letters can be either consonants or vowels. So, each of them have 26 options.
Note that the question asks you to find out the number of distinct initials and not initials where
the letters are distinct.
Hence answer = 26*26*26*21 = 263 * 21

Question:
When four fair dice are rolled simultaneously, in how many outcomes will at least one of the
dice show 3?

(1) 155
(2) 620
(3) 671
(4) 625
Correct Answer - (3)
Solution:

When 4 dice are rolled simultaneously, there will be a total of 6 4 = 1296 outcomes.

The number of outcomes in which none of the 4 dice show 3 will be 54 = 625 outcomes.

Therefore, the number of outcomes in which at least one die will show 3 = 1296 – 625 = 671

Question:
In how many ways can the letters of the word EDUCATION be rearranged so that the relative
position of the vowels and consonants remain the same as in the word EDUCATION?

(1) 9!/4
(2) 9!/(4!*5!)
(3) 4!*5!
(4) None of these
Correct Answer - (3)
Solution:

The word EDUCATION is a 9 letter word, with none of the letters repeating.

The vowels occupy 3, 5, 7th and 8th position in the word and the remaining 5 positions are
occupied by consonants

As the relative position of the vowels and consonants in any arrangement should remain the
same as in the word EDUCATION, the vowels can occupy only the aforementioned 4 places and
the consonants can occupy 1st, 2nd, 4th, 6th and 9th positions.

The 4 vowels can be arranged in the 3rd, 5th, 7th and 8th position in 4! Ways.

Similarly, the 5 consonants can be arranged in 1 st, 2nd, 4th, 6th and 9th position in 5! Ways.

Hence, the total number of ways = 4! * 5!.


Question:
How many ways can 10 letters be posted in 5 post boxes, if each of the post boxes can take
more than 10 letters?

(1) 5^10
(2) 10^5
(3) 10P5
(4) 10C5
Correct Answer - (1)
Solution:

Each of the 10 letters can be posted in any of the 5 boxes.

So, the first letter has 5 options, so does the second letter and so on and so forth for all of the
10 letters.

i.e. 5*5*5*….*5 (upto 10 times)

= 5 ^ 10.

10.Question:
A team of 8 students goes on an excursion, in two cars, of which one can seat 5 and the other
only 4. In how many ways can they travel?

(1) 9
(2) 26
(3) 126
(4) 3920
Correct Answer - (3)
Solution:

There are 8 students and the maximum capacity of the cars together is 9.

We may divide the 8 students as follows

Case I: 5 students in the first car and 3 in the second

Or Case II: 4 students in the first car and 4 in the second


8
Hence, in Case I: 8 students are divided into groups of 5 and 3 in C3 ways.
8
Similarly, in Case II: 8 students are divided into two groups of 4 and 4 in C4 ways.

Therefore, the total number of ways in which 8 students can travel is 8C3 + 8C4 = 56 + 70 = 126
How many ways can 4 prizes be given away to 3 boys, if each boy is eligible for all the prizes?

(1) 256
(2) 12
(3) 81
(4) None of these
Correct Answer - (3)
Solution:

Any one prize can be given to any one of the 3 boys and hence there are 3 ways of distributing
each prize.

Question:
There are 12 yes or no questions. How many ways can these be answered?

(1) 1024
(2) 2048
(3) 4096
(4) 144
Correct Answer - (3)
Solution:

Each of the questions can be answered in 2 ways (yes or no)

Therefore, no. of ways of answering 12 questions = 2 12 = 4096 ways

Hence, the 4 prizes can be distributed in 34= 81 ways.

Question:
How many words can be formed by re-arranging the letters of the word ASCENT such that A and
T occupy the first and last position respectively?

(1) 5!
(2) 4!
(3) 6! - 2!
(4) 6! / 2!
Correct Answer - (2)
Solution:

As A and T should occupy the first and last position, the first and last position can be filled in
only one way. The remaining 4 positions can be filled in 4! Ways by the remaining words
(S,C,E,N,T). hence by rearranging the letters of the word ASCENT we can form 1x4! = 4! words.

Question:
Four dice are rolled simultaneously. What is the number of possible outcomes in which at least
one of the die shows 6?

(1) 6! / 4!
(2) 625
(3) 671
(4) 1296
Correct Answer - (3)
Solution:

When 4 dice are rolled simultaneously, there are 6 4 = 1296 outcomes. The converse of what is
asked in the question is that none of the dice show '6'. That is all four dice show any of the
other 5 numbers. That is possible in 54 = 625 outcomes.

Therefore, in 1296 - 625 = 671 outcomes at least one of the dice will show 6.

Question:
How many alphabets need to be there in a language if one were to make 1 million distinct 3
digit initials using the alphabets of the language?

(1) 26
(2) 50
(3) 100
(4) 1000
Correct Answer - (3)
Solution:

1 million distinct 3 digit initials are needed.

Let the number of required alphabets in the language be ‘n’.

Therefore, using ‘n’ alphabets we can form n * n * n = n 3 distinct 3 digit initials.

Note distinct initials is different from initials where the digits are different.
For instance, AAA and BBB are acceptable combinations in the case of distinct initials while
they are not permitted when the digits of the initials need to be different.

This n3 different initials = 1 million

i.e. n3 = 106 (1 million = 106)


3 2 3
=> n = (10 ) => n = 102 = 100

Hence, the language needs to have a minimum of 100 alphabets to achieve the objective.
Question:
In how many ways can the letters of the word MANAGEMENT be rearranged so that the two As
do not appear together?

(1) 10! - 2!
(2) 9! - 2!
(3) 10! - 9!
(4) None of these
Correct Answer - (4)
Solution:

The word MANAGEMENT is a 10 letter word.

Normally, any 10 letter word can be rearranged in 10! ways.

However, as there are certain letters of the word repeating, we need to account for those. In
this case, the letters A, M, E and N repeat twice each.

Therefore, the number of ways in which the letters of the word MANAGEMENT can be

rearranged reduces to .

The problem requires us to find out the number of outcomes in which the two As do not appear
together.

The number of outcomes in which the two As appear together can be found out by considering
the two As as one single letter. Therefore, there will now be only 9 letters of which three of
them E, N and M repeat twice. So these 9 letters with 3 of them repeating twice can be

rearranged in ways.

Therefore, the required answer in which the two As do not appear next to each other =

Total number of outcomes - the number of outcomes in which the 2 As appear together
=> ways.

There are 5 Rock songs, 6 Carnatic songs and 3 Indi pop songs. How many different albums can
be formed using the above repertoire if the albums should contain at least 1 Rock song and 1
Carnatic song?

(1) 15624
(2) 16384
(3) 6144
(4) 240
Correct Answer - (1)
Solution:

There are 2n ways of choosing ‘n’ objects. For e.g. if n = 3, then the three objects can be
chosen in the following 23 ways - 3C0 ways of choosing none of the three, 3C1 ways of choosing
one out of the three, 3C2 ways of choosing two out of the three and 3C3 ways of choosing all
three.

In the given problem, there are 5 Rock songs. We can choose them in 25 ways. However, as the
problem states that the case where you do not choose a Rock song does not exist (at least one
rock song has to be selected), it can be done in 25 - 1 = 32 - 1 = 31 ways.

Similarly, the 6 Carnatic songs, choosing at least one, can be selected in 26 - 1 = 64 - 1 = 63


ways.

And the 3 Indi pop can be selected in 23 = 8 ways. Here the option of not selecting even one
Indi Pop is allowed.

Therefore, the total number of combinations = 31 * 63 * 8 = 15624

Question:
What is the value of 1*1! + 2*2! + 3!*3! + ............ n*n!,
where n! means n factorial or n(n-1)(n-2)...1

(1) n(n-1)(n-1)!
(2) (n+1)!/(n(n-1))
(3) (n+1)! - n!
(4) (n + 1)! - 1!
Correct Answer - (4)
Solution:

1*1! = (2 -1)*1! = 2*1! - 1*1! = 2! - 1!


2*2! = (3 - 1)*2! = 3*2! - 2! = 3! - 2!
3*3! = (4 - 1)*3! = 4*3! - 3! = 4! - 3!
..
..
..
n*n! = (n+1 - 1)*n! = (n+1)(n!) - n! = (n+1)! - n!

Summing up all these terms, we get (n+1)! - 1!

Question:
How many number of times will the digit ‘7' be written when listing the integers from 1 to
1000?

(1) 271
(2) 300
(3) 252
(4) 304
Correct Answer - (2)
Solution:

7 does not occur in 1000. So we have to count the number of times it appears between 1 and
999. Any number between 1 and 999 can be expressed in the form of xyz where 0 < x, y, z < 9.

1. The numbers in which 7 occurs only once. e.g 7, 17, 78, 217, 743 etc
This means that 7 is one of the digits and the remaining two digits will be any of the other 9
digits (i.e 0 to 9 with the exception of 7)

You have 1*9*9 = 81 such numbers. However, 7 could appear as the first or the second or the
third digit. Therefore, there will be 3*81 = 243 numbers (1-digit, 2-digits and 3- digits) in which
7 will appear only once.

In each of these numbers, 7 is written once. Therefore, 243 times.

2. The numbers in which 7 will appear twice. e.g 772 or 377 or 747 or 77
In these numbers, one of the digits is not 7 and it can be any of the 9 digits ( 0 to 9 with the
exception of 7).
There will be 9 such numbers. However, this digit which is not 7 can appear in the first or
second or the third place. So there are 3 * 9 = 27 such numbers.

In each of these 27 numbers, the digit 7 is written twice. Therefore, 7 is written 54 times.

3. The number in which 7 appears thrice - 777 - 1 number. 7 is written thrice in it.

Therefore, the total number of times the digit 7 is written between 1 and 999 is 243 + 54 + 3 =
300

Pr
Question

What is the probability of getting at least one six in a single throw of three unbiased dice?

(1) 1 / 6
(2) 125 / 216
(3) 1 / 36
(4) 81 / 216
(5) 91 / 216

Correct choice is (5) and Correct Answer is 91 / 216

Explanatory Answer

Every die has got six sides. Each of the sides is numbered from 1 to 6.

When a single unbiased die is thrown you can have six possible outcomes.

When two dice are thrown simultaneously, the total number of outcomes will be 6 * 6 = 36
Similarly, when three dice are thrown simultaneously, the total number of outcomes will be
6*6*6=216.
We need to find out the number of cases in which at least one of the facing sides shows 6.

At least one means - either one dice or two dice or all three.

Case 1: Let us take the easiest case first - all three dice showing '6' - There is only one such
possibility.

Case 2: The number of cases in which two of the dice show 6 and one of them is a different
number.
For eg an event like 6 6 5 will be one of the outcomes for case 2.

As two of the dice show '6' , it can happen in only one way. The third die shows a different number, a
number other than 6, and it can be any of the 5 other numbers. Therefore, there will 5 possible
options i.e. (1, 6, 6), (2, 6, 6), (3, 6, 6), (4, 6, 6), (5, 6, 6).

However, each of these possibilities can have three different arrangements depending upon where
the third different digit appears. For example take (1, 6, 6) case - it will have three options (1, 6, 6),
(6, 1, 6), (6, 6, 1).

Therefore, the total number of events in which 2 of the dice will show '6' and one will show a
different number = 5*3 = 15 cases.

Case 3: When only one of the die shows '6' and the other two show numbers other than '6'.

The die showing '6' has only one option. The other two dice can have any of the '5' options.
Therefore, the total number of possibilities = 1*5*5 = 25.

However, the die showing '6' can either be the first die or the second die or the third die.

Therefore, there are a total of 25 * 3 = 75 possibilities.

Total possible outcomes where at least one die shows '6' = Case 1 + Case 2 + Case 3 = 1 + 15 +75
= 91.

Therefore, the required probability =

Alternate approach:

Find the number of cases in which none of the digits show a '6'.

i.e. all three dice show a number other than '6', 5 * 5 *5 = 125 cases.

Total possible outcomes when three dice are thrown = 216.

The number of outcomes in which at least one die shows a '6' = Total possible outcomes when three
dice are thrown - Number of outcomes in which none of them show '6'.

= 216 - 125 = 91.

The required probability =

Question 4 the day: August 06, 2002


The question for the day is from the topic Probability.
What is the probability that a two digit number selected at random will be a multiple of '3'
and not a multiple of '5'?
(1) 2 / 15 (2) 4 / 15 (3) 1 / 15 (4) 4 / 90
Correct Answer - (2)
Solution:

There are a total of 90 two digit numbers. Every third number will be divisible by '3'. Therefore,
there are 30 of those numbers that are divisible by '3'.

Of these 30 numbers, the numbers that are divisible by '5' are those that are multiples of '15'. i.e.
numbers that are divisible by both '3' and '5'. There are 6 such numbers -- 15, 30, 45, 60, 75 and
90.

We need to find out numbers that are divisible by '3' and not by '5', which will be 30 - 6 = 24.

24 out of the 90 numbers are divisible by '3' and not by '5'.

The required probability is therefore,

Question 4 the day: August 23, 2002


The question for the day is from the topic of probability.
A man bets on number 16 on a roulette wheel 14 times and losses each time. On the 15th
span he does a quick calculation and finds out that the number 12 had appeared twice in
the 14 spans and is therefore, unable to decide whether to bet on 16 or 12 in the 15 th span.
Which will give him the best chance and what are the odds of winning on the bet that he
takes? (Roulette has numbers 1 to 36)
(2 (3 (4
(1) 16; 22 : 14 12; 72 : 1 12; 7 : 1 Either; 35 : 1
) ) )
Correct Answer - (4)

Solution:

Each of the span is an independent event and the outcome of the 15th span will not depend on the
outcome of the earlier spans.

The question for the day is from the topic of Probability


When two dice are thrown simultaneously, what is the probability that the sum of the two
numbers that turn up is less than 11?
(2
(1) 5 / 6 11 / 12
)

(4
(3) 1 / 6 1 / 12
)
Correct Answer - (2)

Solution:

Instead of finding the probability of this event directly, we will find the probability of the non-
occurrence of this event and subtract it from 1 to get the required probability.

Combination whose sum of 12 is (6,6)

Combinations whose sum of 11 is (5,6), (6,5).

Therefore, there are totally 3 occurrences out of 36 occurrences that satisfy the given condition.

Probability whose sum of two numbers is greater than or equal to 11 = 3 / 36 = 1 / 12.

Hence probability whose sum of two numbers is lesser than 11 = 1 - 1 / 12 = 11 / 12.


The question for the day is from the topic of Probability.
When 4 dice are thrown, what is the probability that the same number appears on each of
them?
(2
(1) 1/36 1/18
)

(4
(3) 1/216 1/5
)
Correct Answer - (3)

Solution:

Sample space (Denominator): When 4 dice are thrown simultaneously, then the total number of
possible outcomes is 64 = 1296

Event (Numerator): The chances that all the dice show same number (1,1,1,1), (2,2,2,2), (3,3,3,3),
(4,4,4,4), (5,5,5,5), (6,6,6,6)} is 6.

Probability = Event/ Sample space = 6/64 = 1/63 = 1/216.

Probability - Quant/Math - CAT 2009 View Archives


Question 4 the day: June 23, 2003 1. Algebra
2. Progressions
The question for the day is from the topic of Probability.
An experiment succeeds twice as often as it fails. What is the
3. Averages
probability that in the next 5 trials there will be four 4. Clocks and
successes? Calendars
(2 5. Data Sufficiency
(1) 0 (2/3)^4
) 6. English Grammar
(4 7. Function
(3) 5*((2/3)^4)*(1/3) ((2/3)^4)*(1/3)
) 8. Geometry
Correct Answer - (3) 9. Interest
Solution: 10. Mensuration
11. Mixtures &
An experiment succeeds twice as often as it fails. Alligations
12. Number System
i.e. the probability of its success is 2/3 and the probability of its failure 13. Percentages
is 1/3.
14. Permutation &
In the next 5 trials the experiment needs to succeed in 4 out of the 5 Combination
trials. 15. Pipes & Cisterns
And Work & Time
4 out of the 5 trials in which it succeeds could be selected in 5C4 ways 16. Probability
= 5 ways.
17. Profit & Loss
And as 4 of them are successes, they have a probability of 2/3 and the 18. Races
one that is a failure will have a probability of 1/3. 19. Ratio, Proportion
Hence, the required probability = 5*((2/3)^4)*(1/3) 20. Speed, Time &
Distance
21. Trigonometry
22. Miscellaneous

23. General
Knowledge
The question for the day is from the topic of Probability.
A number is selected at random from first thirty natural numbers. What is the chance that it
is a multiple of either 3 or 13?

(2
(1) 17/30 2/5
)

(4
(3) 11/30 4/15
)
Correct Answer - (2)

Solution:

The probability that the number is a multiple of 3 is 10/30. (Since 3*10 = 30).

Similarly the probability that the number is a multiple of 13 is 2/30. {Since 13*2 = 26).

Neither 3 nor 13 has common multiple from 1 to 30. Hence these events are mutually exclusive
events. Therefore chance that the selected number is a multiple of 3 or 13 is (10+2)/30 = 2/5

The question for the day is from the topic Probability.

A man can hit a target once in 4 shots. If he fires 4 shots in succession, what is the probability that
he will hit his target?

(1) 1 (2)

(3) (4)

Correct choice - (4) Correct Answer -( )

Explanatory Answer

The man will hit the target even if he hits it once or twice or thrice or all four times in the four shots
that he takes.

So, the only case where the man will not hit the target is when he fails to hit the target even in one
of the four shots that he takes.

The probability that he will not hit the target in one shot = 1 - =

Therefore, the probability that he will not hit the target in all the four shots =

Hence, the probability that he will hit the target at least in one of the four shots =
1 -

= .

In
Question
Which of the following inequalities have a finite range of values of "x" satisfying them?

1. x2 + 5x + 6 > 0
2. |x + 2| > 4
3. 9x - 7 < 3x + 14
4. x2 - 4x + 3 < 0

Correct Answer - x2 - 4x + 3 < 0. Choice (4)


Explanatory Answer

We have to find out the values of "x" that will satisfy the four inequalities given in the answer
choices and check out the choice in which the range of values satisfying is finite.

Choice 1
Factorizing the given equation, we get (x + 2)(x + 3) > 0.
This inequality will hold good when both x + 2 and x + 3 are simultaneously positive or
simultaneously negative.

Evaluating both the options, we get the range of values of "x" that satisfy this inequality to be x < -2
or x > -3. i.e., "x" does not lie between -2 and -3 or an infinite range of values.

Choice 2
|x + 2| > 4 is a modulus function and therefore, has two options
Option 1: x + 2 > 4 or
Option 2: (x + 2) < -4.
Evaluating the two options we get the values of "x" satisfying the inequality as x > 2 and x < -6. i.e.,
"x" does not lie between -6 and 2 or an infinite range of values.

Choice 3
9x - 7 < 3x + 14
Simplifying, we get 6x < 21 or x < 3.5. Again an infinite range of values.

Choice 4
x^2 - 4x + 3 < 0
Factorizing we get, (x - 3)(x - 1) < 0.

This inequality will hold good when one of the terms (x - 3) and (x - 1) is positive and the other is
negative.
Evaluating both the options, we get 1 < x < 3. i.e., a finite range of values for "x".

Hence, choice 4 is the correct answer.


Question

For what range of values of 'x' will the inequality 15x - > 1?

(1) x > 0.4

(2) x <
(3) - < x < 0.4, x >

(4) - < x < 0, x >

Correct choice (4). Correct Answer - (- < x < 0, x > )


Explanatory Answer

We can rewrite the above inequality as 15x - - 1 > 0

i.e., > 0

or > 0.

Factorizing we get, > 0


The above inequality will hold good if the numerator and denominator are both positive or are both
negative.

Case 1: When (5x - 2)(3x + 1) > 0 and x > 0

This will hold true for values of 'x' that do not lie between and and for x > 0.

Combining all these conditions, we get x >

Case 2: When (5x - 2)(3x + 1) < 0 and x < 0

This will hold true for the following values of 'x': < x < and x < 0.

Combining, we get < x < 0.

Therefore, the final range for which the above inequality will hold true is given by < x < 0 and x

> .
Question
Find the range of real values of x satisfying the inequalities 3x - 2 > 7 and 4x - 13 > 15.

(1) x > 3
(2) x > 7
(3) x < 7
(4) x < 3

Correct Choice is (2) and Correct Answer is x > 7


Explanatory Answer
3x - 2 > 7 => 3x > 9 => x > 3
and 4x - 13 > 15 => 4x > 28 => x > 7, 7 is greater than 3.
Therefore, x > 7.
The question for the day is from the topic of Inequalities.

Question
Solve the inequality 33x-2 > 1

(1) x > 1
(2) x > 3
(3) x > 2/3
(4) x > 1/3

Correct Choice is (3) and Correct Answer x > 2/3


Explanatory Answer

33x-2 > 1
33x-2 > 30
=> 3x - 2 > 0
3x > 2 => x > 2/3

Question

Solve for real 'x' if :

(1) x > 9
(2) 0 < x < 9
(3) x < 0
(4) None of these

Correct Choice is (2) and Correct Answer is 0 < x < 9


Explanatory Answer

Squaring on both sides we get, 9x-x2 > 0

=> x2-9x < 0

=> x(x-9) < 0

=> 0 < x < 9

Da
The question for the day is a data sufficiency question.
In a data sufficiency question you will be given a question that will be followed by two
statements. You are expected to answer if the information provided in the two statements is
sufficient to answer the given question.

Please be careful to read the instructions before the data sufficiency questions. These
instructions have changed from CAT to CAT. CAT 2003 (Re test) had a different set as compared
to the ones seen in the previous three or four Common Admission Tests.
Directions
This data sufficiency problem consists of a question and two statements, labeled (A) and (B), in
which certain data are given. You have to decide whether the data given in the statements are
sufficient for answering the question. Using the data given in the statements, plus your
knowledge of mathematics and everyday facts (such as the number of days in a leap year or the
meaning of the word counterclockwise), you must indicate whether -
(1) The question can be answered using either statement (A) alone but not statement
(B) or statement (B) alone but not statement (A).
(2) If either of the two statements individually is sufficient to answer the question
(3) If both statements (A) and (B) together are required to answer the question
(4) The question cannot be answer despite using the information provided in
statements (A) and (B) taken together.
Question
How long will it take for two pipes A and B to fill an empty cistern if they worked alternately
for an hour each?
A. Working alone, Pipe A can fill the cistern in 40 hours
B. Pipe B is one third as efficient as Pipe A
Correct choice (3). Correct Answer - (Both the statements together are sufficient to answer
the question.)
Explanatory Answer:

From statement A, we know that Pipe A can fill the tank in 40 hours. However, this information
is not sufficient as we do not have the data for Pipe B. Hence, statement A alone cannot
answer the given question.

From statement B, we know that Pipe B is one third as efficient as pipe A. However, we do not
know the rate at which Pipe A fills the tank. Hence, we will not be able to find the rate at
which Pipe B fills the cistern. Therefore, statement B alone is not sufficient to answer the
question.

Now, if we combine the two statements, we know that Pipe A take 40 hours to fill the cistern.
Pipe B takes 120 hours to fill the cistern.

If they worked alternately, then either Pipe A could have started the cycle or Pipe B could have
started the cycle.

If Pipe A started the sequence of filling alternately, then at the end of two hours, the two

pipes together would have filled th of the tank in an hour. Or the cistern will fill
in 30 hours.

If Pipe B started the sequence, then at the end of 2 hours, the two pipes together would have

filled th of the tank in an hour. Or the cistern will fill in 30 hours.

As the answer obtained irrespective of which pipe started the sequence is the same, the
correct answer is (3) - i.e., both the statement are required to answer the question

What is the value of X, if X and Y are two distinct integers and their product is 30?
I. X is an odd integer
II. X > Y
Correct choice (4). Correct Answer - (The value of X cannot be determined from the
information provided)
Solution:

From the question, we know that both X and Y are distinct integers and their product is 30.

30 can be obtained as a product of two distinct integers in the following manner


1 * 30 (-1) * (-30)
2 * 15 (-2) * (-15)
3 * 10 (-3) * (-10)
5 * 6 (-5) * (-6)
Statement I: From this statement, we know that the value of X is odd. Therefore, X can be one
of the following values: 1, -1, 3, -3, 5, -5. So, using the information in statement I we will not
be able to conclusively decide the value of X. Hence, statement I alone is not sufficient to
answer the question.

Statement II: From this statement, we know that the value of X > Y. From the given
combinations, X can take more than one value. Hence, using the information in statement II,
we will not be able to find the value of X.

Combining the two statements, we know that X is odd and that the value of X > Y.
The combinations that satisfy both the conditions include X taking the value of -1, -3 and -5.

As the information provided in the two statements independently or together are not sufficient
to answer the question, the answer choice is (4).

The question for the day is from the topic of Data Sufficiency
The set S of numbers has the following properties: I) If x is in S, then 1/x is in S. II) If both
x and y are in S, then so is x + y. Is 3 in S? (1) 1/3 is in S. (2) 1 is in S.
Mark 1 If the question can be answered by
Mark 2 If the question can be
using one of the statements alone, but cannot (2
(1) answered by using either
be answered by using the other statement )
statement alone.
alone.

Mark 3 If the question can be answered by Mark 4 If the question cannot


(4
(3) using both statements together, but cannot be answered even by using both
)
be answered by using either statement alone. the statements together.
Correct Answer - (2)
Solution:

Consider (1) alone. Since 1/3 is in S, we know from Property I that 1/(1/3) = 3 is in S. Hence, (1) is
sufficient.

Consider (2) alone. Since 1 is in S, we know from Property II that 1 + 1 = 2 (Note, nothing in
Property II prevents x and y from standing for the same number. In this case both stand for 1.) is
in S. Applying Property II again shows that 1 + 2 = 3 is in S. Hence, (2) is also sufficient.
Data Sufficiency - Quant/Math - CAT 2009 View Archives
Question 4 the day: August 05, 2003 1. Algebra
2. Progressions
The question for the day is from the topic of Data Sufficiency.
Mark 1 If the question can be answered by using one of the
3. Averages
statements alone, but cannot be answered by using the other 4. Clocks and
statement alone. Calendars
Mark 2 If the question can be answered by using either 5. Data Sufficiency
statement alone.
Mark 3 If the question can be answered by using both
6. English Grammar
statements together, but cannot be answered by using either 7. Function
statement alone. 8. Geometry
Mark 4 If the question cannot be answered even by using
both the statements together.
9. Interest
10. Mensuration
Is x = y? 11. Mixtures &
Alligations
12. Number System
A. = 4 13. Percentages
B. (x – 50)2 = (y – 50)2
14. Permutation &
(2 Combination
(1) 1 4
)
15. Pipes & Cisterns
And Work & Time
16. Probability
17. Profit & Loss
18. Races
19. Ratio, Proportion
(4
(3) 3 2
)
Correct Answer - (1)
Solution:

Statement A. = 4 => 1 + 1 + = 4

=> = 2. This is possible when x = y. Hence statement A alone is sufficient.

Statement B. (x – 50)2 = (y – 50)2 . We cannot say if x = y in this case. Take for example, let x =
100 and y = 0. Then (x – 50)2 = (100 – 50)2 = 502 = 2500.

And (y – 50)2 = (0 – 50)2 = 502 = 2500.

Hence, statement A alone is sufficient and statement B alone is not sufficient.

The question for the day is from the topic of Data Sufficiency.
Is the smallest of five consecutive integers even?

A. The product of the five integers is 0

B. The arithmetic mean of the five integers is 0.

(2
(1) 1 2
)

(4
(3) 3 4
)
Correct Answer - (1)
Solution:

If the smallest of five consecutive integers is even, then the first, third and fifth integers will be
even. From statement A, we know that one of the 5 numbers is 0. However, we will not be able to
say which of the 5 numbers happen to be 0.

From statement B, we know the arithmetic mean of the 5 numbers is 0. The A.M of five consecutive
integers is the third integer, which is 0. 0 is even. Hence, the smallest of the 5 consecutive integers
is even. Hence statement B alone is sufficient and the answer is (1).

The question for the day is from the topic of Data Sufficiency.

Choose 1 if the question can be answered by one of the statements alone and not by the
other.

Choose 2 if the question can be answered by using either statement alone.

Choose 3 if the question can be answered by using both the statements together, but
cannot be answered by using either statement alone.

Choose 4 if the question cannot be answered even by using both statements together.

Is X a prime number, given that X is a positive integer?

A. X^4 > 3000


B. X^4 < 10,000

(2
(1) 1 2
)

(4
(3) 3 4
)
Correct Answer - (3)
Solution:

From statement A we know that X 4 is greater than 3000. There are infinite values for X possible for
which X4 is greater than 3000

From statement B we know that X4 is less than 10,000. This one is a lot better. There are only 9
integer values that satisfy this condition. But this still does not give us a unique answer.

Combining the two statements, we know that 2000 < X4 < 10000. The following values of X satisfy
this condition -> 8 and 9. Both 8 and 9 are not prime. Hence, the question can be answered
conclusively using the two statements. Hence answer choice (3)

Question 4 the day: March 30, 2004

The question for the day is a data sufficiency question.

About 5 to 10 questions in the quantitative section / Data Interpretation section of the CAT is of
Data sufficiency type.

In a data sufficiency question you will be given a question that will be followed by two statements.
You are expected to answer if the information provided in the two statements is sufficient to answer
the given question.

Please be careful to read the instructions before the data sufficiency questions. These instructions
have changed from CAT to CAT. CAT 2003 (Re test) had a different set as compared to the ones
seen in the previous three or four Common Admission Tests.

Directions
This data sufficiency problem consists of a question and two statements, labeled (A) and (B), in
which certain data are given. You have to decide whether the data given in the statements are
sufficient for answering the question. Using the data given in the statements, plus your knowledge
of mathematics and everyday facts (such as the number of days in a leap year or the meaning of
the word counterclockwise), you must indicate whether —
(1) The question can be answered using either statement (A) alone but not statement (B) or
statement (B) alone but not statement (A).

(2) If either of the two statements individually is sufficient to answer the question

(3) If both statements (A) and (B) together are required to answer the question

(4) The question cannot be answer despite using the information provided in statements (A) and
(B) taken together.

Question
Is m divisible by 6?

(A) m is divisible by 3
(B) m is divisible by 4

Correct choice - (3)


Explanatory answer

We need to answer if m is divisible by 6. The answer has to be a definitive YES or a NO.

The test of divisibility for 6 is that the number should be divisible by both 3 and 2.

From statement (A) we know that m is divisible by 3. However, this does not answer the question if
m is also divisible by 2. Hence, statement (A) alone is not sufficient. We can rule out answer choices
(2). The correct answer has to be between (1), (3) or (4).

From statement (B) we know that m is divisibly by 4. If m is divisible by 4, then m should surely be
divisible by 2. However, from statement (B) alone we do not know if m is divisible by 3. Therefore,
statement (B) alone is also not sufficient. Hence, we can eliminate answer choice (1).

Combining the two statements, we know that m is divisible by 3 and by 4. Hence, we can conclude
that m is divisible by 6. Choice (3 ) is correct.
Question 4 the day: May 24, 2004

The question for the day is a data sufficiency question.


About 5 to 10 questions in the quantitative section / Data Interpretation section of the
CAT is of Data sufficiency type.

Instructions

This data sufficiency problem consists of a question and two statements, labeled (A) and
(B), in which certain data are given. You have to decide whether the data given in the
statements are sufficient for answering the question. Using the data given in the
statements, plus your knowledge of mathematics and everyday facts (such as the
number of days in a leap year or the meaning of the word counterclockwise), you must
indicate whether
(1) The question can be answered using (2 If either of the two statements
either statement (A) alone but not ) individually is sufficient to answer
statement (B) or statement (B) alone the question
but not statement (A).

(3) If both statements (A) and (B) (4 The question cannot be answer
together are required to answer the ) despite using the information
question provided in statements (A) and
(B) taken together.

Note:

In data sufficient problems that ask for the value of a quantity the data given in the statements
are sufficient only when it is possible to determine exactly one numerical value for the quantity.

If a salesman received a commission of 3% of the sales that he has booked in a month, what was
the sales booked by the salesman in the month of November 2003?

(A) The sales booked by the salesman in the month of November 2003 minus salesman’s
commission was Rs.245,000
(B) The selling price of the sales booked by the salesman in the month of November 2003 were
125 percent of the original purchase price of Rs.225,000.
Correct choice - (2)

Explanatory answer

From statement (1), we know that the sales value after the salesman’s commission. If his
commission is 3% of the sales booked. Then the net sales value is 100 – 3 = 97% of the sales
booked.
From statement (1), we know that 97% of sales booked = Rs.245,000. So we can find out the
sales booked. Statement (1) alone is sufficient.

From statement (2), we know that the original cost of the products is Rs.225,000. We know the
sales booked = 1.25 * 225,000. Hence, statement (2) is also sufficient.

As each of the two statements are independently sufficient to answer the question, choice (2) is
the best answer.

Home | Question Bank | Top B-Schools | US B-Schools | GD-PI | Forums


| Bargain | Doubt Fire
©

You might also like